Can the intersection of nested closed intervals be empty?

an upper bound, then there exists an n>m such that an>bm, contradicting the nested intervals hypothesis.
  • #1
kingwinner
1,270
0

Homework Statement


Nested interval theorem:
Suppose [an,bn] is a nested sequence of closed intervals, i.e. [an+1,bn+1] is contained in [an,bn] for all n≥1. Then the intersection of all these intervals is nonempty.

Proof:
an is an increasing sequence bounded above by b1 (or any bn), so sup an = a exists and a≤bm for all m.
So "a" is a lower bound of bm
Thus let b = inf bm, then we have a≤b (since any lower bound is ≤ its greatest lower bound).
Then [a,b] is contained in [an,bn] for all n since an≤a and b≤bn for all n (because a is an upper bound of an and b is a lower bound of bn)
Therefore, the intersection is nonempty.
===========================

1) In this theorem, it says that "...the seqeunce of CLOSED intervals...". Is it enough to have closed intervals, or must the endpoints also be FINITE numbers? Can any of the endpoints be -∞ or +∞? (reacall that intervals like [a,∞) are classified as closed interval)

2) In the proof, I am confused and I really don't understand why "a≤bm for all m". Can somebody please explain this?

Homework Equations


N/A

The Attempt at a Solution


N/A

Thank you! :)
 
Physics news on Phys.org
  • #2
kingwinner said:
1) In this theorem, it says that "...the seqeunce of CLOSED intervals...". Is it enough to have closed intervals, or must the endpoints also be FINITE numbers? Can any of the endpoints be -∞ or +∞? (reacall that intervals like [a,∞) are classified as closed interval)

[tex] \bigcap_{n=1}^\infty [n,\infty)=\emptyset[/tex]
 
Last edited:
  • #3
kingwinner said:
2) In the proof, I am confused and I really don't understand why "a≤bm for all m". Can somebody please explain this?

bm is an upper bound for the sequence (an). By definition of the supremum, sup an is less than or equal to bm
 
  • #4
iomtt6076 said:
[tex] \bigcap_{n=1}^\infty [n,\infty)=\emptyset[/tex]

But [n,∞) satisifies ALL assumptions as stated in the theorem (it is a nested sequence of closed intervals). So is the theorem wrong? What should be the correct statement of the theorem?
 
  • #5
kingwinner said:
But [n,∞) satisifies ALL assumptions as stated in the theorem (it is a nested sequence of closed intervals). So is the theorem wrong? What should be the correct statement of the theorem?

I think it should be "... a sequence of closed, bounded, non-empty intervals in [tex]\mathbb{R}[/tex]..."
 
  • #6
iomtt6076 said:
bm is an upper bound for the sequence (an). By definition of the supremum, sup an is less than or equal to bm

I agree that for each m, bm is an upper bound for the sequence (an).
But I still don't understand why "sup an≤bm for all m."
By definition of upper bound, sup an≥an for all n, but this idea seems to be unconnected with the above and I don't see how sup an can possibly be related to bm. They are not even talking about the same sequence.

Can someone please explain this in more detail?

Thanks!
 
  • #7
kingwinner said:
I agree that for each m, bm is an upper bound for the sequence (an).
But I still don't understand why "sup an≤bm for all m."
By definition of upper bound, sup an≥an for all n, but this idea seems to be unconnected with the above and I don't see how sup an can possibly be related to bm. They are not even talking about the same sequence.

Can someone please explain this in more detail?

Thanks!

Definition:
A number s is the supremum for a sequence if :
1) s is an upper bound for the sequence
2) if p is any upper bound for the sequence, s is less than or equal to p

so we are talking about 2) here. bm is an upper bound for the sequence; thus by definition sup an<=bm. It doesn't matter whether or not bm is related to a sequence. All that matters is whether it is an upper bound. If this isn't good enough, then what about this: sup an is the least upper bound. suppose for contradiction bm<sup an. remember that bm is an upper bound for an. but then sup an wouldn't be the least upper bound because there is another upper bound, bm, that is less than it. so we must have bm>=sup an
 
  • #8
iomtt6076 said:
Definition:
A number s is the supremum for a sequence if :
1) s is an upper bound for the sequence
2) if p is any upper bound for the sequence, s is less than or equal to p

so we are talking about 2) here. bm is an upper bound for the sequence; thus by definition sup an<=bm. It doesn't matter whether or not bm is related to a sequence. All that matters is whether it is an upper bound.


If this isn't good enough, then what about this: sup an is the least upper bound. suppose for contradiction bm<sup an. remember that bm is an upper bound for an. but then sup an wouldn't be the least upper bound because there is another upper bound, bm, that is less than it. so we must have bm>=sup an

Thanks, so it just follows from the fact that the least upper bound of (a_n) is less than or equal to any upper bound of (a_n). Got it!:)

One more question...

How can we PROVE that bm is an upper bound for the sequence (an) for ALL m=1,2,3,...?

When I draw a picture, I can see that this is the case, but I am not sure how to write it out and PROVE rigorously.
We know that
a1≤a2≤a3≤...
and b1≥b2≥b3≥...
But from here on, I don't see why it follows that bm is an upper bound for the sequence (an) for ALL m=1,2,3,... .

Thanks!
 
  • #9
kingwinner said:
Thanks, so it just follows from the fact that the least upper bound of (a_n) is less than or equal to any upper bound of (a_n). Got it!:)

One more question...

How can we PROVE that bm is an upper bound for the sequence (an) for ALL m=1,2,3,...?

When I draw a picture, I can see that this is the case, but I am not sure how to write it out and PROVE rigorously.
We know that
a1≤a2≤a3≤...
and b1≥b2≥b3≥...
But from here on, I don't see why it follows that bm is an upper bound for the sequence (an) for ALL m=1,2,3,... .

Thanks!

I think it comes from the hypothesis of nested intervals. If there exists an m such that bm is not an upper bound, then ak>bm for some k. So [ak,bk] would be disjoint from [am,bm], which contradicts the assumption of nested intervals. When you have a sequence of nested intervals, no two intervals can be disjoint.
 
  • #10
Makes sense. Thank you! :)
 
  • #11
Another proof:
(an) is an increasing sequence bounded above (by b1), so
lim an = sup an = a
n->∞

(bn) is a decreasing sequence bounded below (by a1), so
lim bn = inf bn = b
n->∞

an≤bn for all n
=> a ≤ b


Thus, ak ≤ a ≤ b ≤ bk for all k≥1. So the point a belongs to the intersection and the intersection is nonempty.
==========================

Why is the red part true? How can we prove it?

Thanks!
 
  • #12
Any of the b_m is an upper bound for the set of all a_n, so the LEAST upper bound a must be less than or equal to b_m for any m. Now you figure out why b must in turn be greater than or equal to a.
 
  • #13
Yes, I already understood that a ≤ bm for all m. It just follows from the fact that the least upper bound of (a_n) is less than or equal to any upper bound of (a_n). (see post #8)


And now my problem is about the second proof.

How can we prove that "an ≤ bn for all n IMPLIES lim an ≤ lim bn"?

It seems clear in my mind, but how can we prove rigorously using epsilon?

Thanks!
 
  • #14
If it helps, here's how I might tackle this problem. You already know that [itex]a_n \leq b_m[/itex] for all [itex]m,n \in \mathbb{N}[/itex]. From this we can conclude that the sequence [itex]\{a_n\}[/itex] has a least upper bound which we denote [itex]\alpha[/itex]. Similarly, the sequence [itex]\{b_n\}[/itex] has a greatest lower bound which we denote [itex]\beta[/itex]. Now, from this we have only three possible scenarios:
  • [tex]a_n \leq \alpha < \beta \leq b_n[/tex]
  • [tex]a_n \leq \alpha = \beta \leq b_n[/tex]
  • [tex]a_n \leq \beta < \alpha \leq b_n[/tex]
If either of the first two cases hold, we know that the theorem we're trying to prove is true. If the third case is true however, we know that the intersection of all intervals [itex][a_n,b_n][/itex] is necessarily empty. Thus, if you can rule the third case out, then you'll have proved the theorem. So, suppose that the third case is true and let [itex]\alpha - \beta = 2\varepsilon > 0[/itex]. Can you derive a contradiction from this?
 
Last edited:

1. What is the Nested Interval Theorem?

The Nested Interval Theorem is a mathematical theorem that states that if you have an infinite sequence of closed intervals, where each interval contains the next one, then the intersection of all these intervals will contain exactly one point.

2. Who discovered the Nested Interval Theorem?

The Nested Interval Theorem was first proven by the Greek mathematician Eudoxus of Cnidus in the 4th century BC. However, it was later refined and generalized by other mathematicians, including Georg Cantor and Karl Weierstrass.

3. What is the significance of the Nested Interval Theorem?

The Nested Interval Theorem is an important result in real analysis and topology. It is used to prove the existence of real numbers and to establish the completeness of the real number system. It is also a fundamental tool for proving other mathematical theorems.

4. Can you give an example of how the Nested Interval Theorem is applied?

One example of applying the Nested Interval Theorem is in the proof of the intermediate value theorem. This theorem states that if a continuous function takes on two different values at two points, then it must also take on every value in between those two points. The Nested Interval Theorem is used to show that there exists a point where the function takes on a specific value within a given interval.

5. Are there any limitations or exceptions to the Nested Interval Theorem?

While the Nested Interval Theorem is a powerful tool, it does have some limitations. For example, it only applies to closed intervals and cannot be extended to open or half-open intervals. Additionally, the Nested Interval Theorem is a theoretical concept and may not always have practical applications in real-world situations.

Similar threads

  • Calculus and Beyond Homework Help
Replies
3
Views
281
  • Calculus and Beyond Homework Help
Replies
14
Views
2K
  • Calculus and Beyond Homework Help
Replies
5
Views
1K
Replies
3
Views
9K
  • Calculus and Beyond Homework Help
Replies
3
Views
2K
  • Calculus and Beyond Homework Help
Replies
3
Views
953
  • Calculus and Beyond Homework Help
Replies
12
Views
1K
  • Calculus and Beyond Homework Help
Replies
32
Views
2K
Replies
1
Views
1K
Back
Top